1003124303

Podobszar: 
Część: 
Project ID: 
1003124303
Accepted: 
1
Clonable: 
0
Easy: 
0
Która z podanych wartości liczby rzeczywistej \( a \), \( b\in\left(0;\frac{\pi}2\right) \) tak, aby \( a < b \), sprawia, że równość \( \int\limits_a^b \cos x\,\mathrm{d}x=2\cos\frac{\pi}4\cdot\sin\frac{\pi}{12} \) jest prawdziwa?
\( a=\frac{\pi}6 \), \( b=\frac{\pi}3 \)
\( a=\frac{\pi}3 \), \( b=\frac{\pi}6 \)
\( a=\frac{\pi}3 \), \( b=\frac{\pi}4 \)
\( a=\frac{\pi}4 \), \( b=\frac{\pi}3 \)